Question

Based on the following information, make an estimate of the stock's beta: Month 1 = Stock...

Based on the following information, make an estimate of the stock's beta: Month 1 = Stock +1.1%, Market +1.5%; Month 2 = Stock +1.4%, Market +2.4%; Month 3 = Stock -2.1%, Market -2.9%. A. Beta is greater than 1.0. B. Beta is less than 1.0. C. Beta equals 1.0. D. There is no consistent pattern of returns.

Homework Answers

Answer #1

Beta is the measure of volatility of returns of a stock in relation to the market. A beta of 1 denotes that stock return is equal to the market return. In month 1, the stock return increases by 1.1% which is less than the increase in market return of 1.4%. Same is the case for month 2. For month 3, the stock decreases by -2.1% which is less than the decrease in market return of -2.9%. From these returns, we can observe that, Beta is less than 1 (or less than market return). (Option B)

Know the answer?
Your Answer:

Post as a guest

Your Name:

What's your source?

Earn Coins

Coins can be redeemed for fabulous gifts.

Not the answer you're looking for?
Ask your own homework help question
Similar Questions
Based on the following information, make an estimate of the stock's beta: Month 1 = Stock...
Based on the following information, make an estimate of the stock's beta: Month 1 = Stock +1.1%, Market +1.5%; Month 2 = Stock +1.4%, Market +2.4%; Month 3 = Stock -2.1%, Market -2.9%. A. Beta is greater than 1.0. B. Beta is less than 1.0. C. Beta equals 1.0. D. There is no consistent pattern of returns.
Estimate a stock's beta based on the following information: Month 1 = Stock + 0.5%, Market...
Estimate a stock's beta based on the following information: Month 1 = Stock + 0.5%, Market + 1.1%; Month 2 = Stock + 1.0%, Market + 1.4%; Month 3 = Stock − 1.5%, Market − 2.0%. Equal to 1.0 Less than 1.0 Greater than 1.0 Indeterminate
BETA AND REQUIRED RATE OF RETURN A stock has a required return of 12%; the risk-free...
BETA AND REQUIRED RATE OF RETURN A stock has a required return of 12%; the risk-free rate is 7%; and the market risk premium is 3%. What is the stock's beta? Round your answer to two decimal places. If the market risk premium increased to 7%, what would happen to the stock's required rate of return? Assume that the risk-free rate and the beta remain unchanged. If the stock's beta is equal to 1.0, then the change in required rate...
Beta and required rate of return A stock has a required return of 11%; the risk-free...
Beta and required rate of return A stock has a required return of 11%; the risk-free rate is 7%; and the market risk premium is 3%. What is the stock's beta? Round your answer to two decimal places. If the market risk premium increased to 9%, what would happen to the stock's required rate of return? Assume that the risk-free rate and the beta remain unchanged. If the stock's beta is greater than 1.0, then the change in required rate...
BETA AND REQUIRED RATE OF RETURN a. A stock has a required return of 11%; the...
BETA AND REQUIRED RATE OF RETURN a. A stock has a required return of 11%; the risk-free rate is 5%; and the market risk premium is 5%. What is the stock's beta? Round your answer to two decimal places. b. If the market risk premium increased to 10%, what would happen to the stock's required rate of return? Assume that the risk-free rate and the beta remain unchanged. If the stock's beta is less than 1.0, then the change in...
Problem 8-5 Beta and required rate of return A stock has a required return of 11%;...
Problem 8-5 Beta and required rate of return A stock has a required return of 11%; the risk-free rate is 6.5%; and the market risk premium is 4%. What is the stock's beta? Round your answer to two decimal places. If the market risk premium increased to 9%, what would happen to the stock's required rate of return? Assume that the risk-free rate and the beta remain unchanged. If the stock's beta is equal to 1.0, then the change in...
Which of the following statements about the beta coefficient is false? A A stock’s beta coefficient...
Which of the following statements about the beta coefficient is false? A A stock’s beta coefficient measures its volatility relative to the market portfolio. B A stock’s beta coefficient can be estimated by plotting the stock’s returns versus the market portfolio’s returns. C A stock’s reported beta coefficient is based on forecasted future volatility. D A stock with a beta coefficient greater than 1.0 is said to be riskier than the market portfolio. E Using the capital asset pricing model,...
A stock has a required return of 12%, the risk-free rate is 3%, and the market...
A stock has a required return of 12%, the risk-free rate is 3%, and the market risk premium is 3%. What is the stock's beta? Round your answer to two decimal places. If the market risk premium increased to 10%, what would happen to the stock's required rate of return? Assume that the risk-free rate and the beta remain unchanged. Do not round intermediate calculations. Round your answer to two decimal places. If the stock's beta is equal to 1.0,...
A stock has a required return of 14%, the risk-free rate is 7.5%, and the market...
A stock has a required return of 14%, the risk-free rate is 7.5%, and the market risk premium is 3%. What is the stock's beta? Round your answer to two decimal places. If the market risk premium increased to 6%, what would happen to the stock's required rate of return? Assume that the risk-free rate and the beta remain unchanged. Do not round intermediate calculations. Round your answer to two decimal places. If the stock's beta is less than 1.0,...
A stock has a required return of 16%, the risk-free rate is 5%, and the market...
A stock has a required return of 16%, the risk-free rate is 5%, and the market risk premium is 3%. What is the stock's beta? Round your answer to two decimal places. If the market risk premium increased to 10%, what would happen to the stock's required rate of return? Assume that the risk-free rate and the beta remain unchanged. Do not round intermediate calculations. Round your answer to two decimal places. If the stock's beta is greater than 1.0,...